LSAT and Law School Admissions Forum

Get expert LSAT preparation and law school admissions advice from PowerScore Test Preparation.

 jadaws
  • Posts: 6
  • Joined: Mar 25, 2013
|
#8690
I'm not sure if this is true, but I have a questions that deals with scheduling 9 patients (LMOPRSTVX) during a 7 day week.

Question 12.2 (pg.3-37) asks....
Which one of the following statements about the doctor's schedule must be true?
So the correct answer is C...3 patients for Friday

Is there a mathematical approach that can be used to attack these questions if there is no rule stating that "Friday can only have X amount of patients"? Hopefully this isn't too confusing. The explanation on pg. 3-117 shows this (3-1-1-1-1-1-1), which basically looks like these types of problems can be solved quickly just by looking at the global rules and making sure that everything adds up to, in this case, 9.
User avatar
 Dave Killoran
PowerScore Staff
  • PowerScore Staff
  • Posts: 5852
  • Joined: Mar 25, 2011
|
#8691
Hi J,

I think I understand what you're getting at, and the answer is yes. In creating the distributions, one thing to note that they are primarily unfixed. That is, the only day that is restricted in the rules is Wednesday. Thus, when looking at the question, as long as Wednesday isn't referred to as the day with 3, you can use the most favorable distribution (in this case, 3-1-1-1-1-1-1) to answer the question. In this sense, questions that rely on distributions are almost purely mathematical.

Whenever you see a game that is based on multiple distributions or an unusual distribution, look for questions of that nature, and then capitalize on them.

Please let me know if this helps. Thanks!

Get the most out of your LSAT Prep Plus subscription.

Analyze and track your performance with our Testing and Analytics Package.